PPI Summative Exam

Ace your homework & exams now with Quizwiz!

A 12-year-old male is brought to the clinic by is mother for what she describes as a "swollen face." His recent medical history is significant for impetigo two weeks ago. On PE he has elevated blood pressure for his age. What is the most appropriate next step in the management? a. Urinalysis b. Antistreptolysin O titers c. Renal ultrasound d. Abdominal CT e. Empiric antibiotics

a. Urinalysis - this is acute post-streptococcal glomerulonephritis (PSGN) - UA is cheap and fast

A 31-year-old G0P1 with a history of hypertension presents to the ED because she believes that she is in labor. She is in her 38th week of pregnancy and her course has thus far been uncomplicated. This morning she began feeling painful contractions and noted vaginal bleeding. She is experiencing lower abdominal and pelvic pain between contractions as well. On exam, she is afebrile, BP is 145/85 mmHg, HR is 102 bpm, and RR is 16 rpm. You note a gravid, hypertonic uterus on exam and moderate blood in the vaginal vault. Ultrasound reveals no abnormalities. Which of the following is the most likely diagnosis? a. Abruptio placentae b. Vasa previa c. Placenta previa d. Chorioamnionitis e. Onset of normal labor

a. Abruptio placentae - history of hypertension and painful antepartum hemorrhage with a normal ultrasound

A 78-year-old man is brought in to the ED by ambulance after his wife noticed that he began slurring his speech and had developed facial asymmetry during dinner approximately 30 minutes ago. His PMH is remarkable only for hypertension. On PE, vital signs are within normal limits except for a heart rate of 105 bpm. He is noted to have distinct right facial paralysis. A non-contrast head CT is performed and is shown. What is the most likely diagnosis in this case? a. Acute hemorrhagic stroke b. Acute ischemic stroke c. Glioblastoma multiforme d. Subdural hematoma e. Partial seizure

a. Acute hemorrhagic stroke - The patient has acute right-sided facial paralysis and hyperdensity on non-contrast CT of the head consistent with acute hemorrhagic stroke

A 61-year-old male complains of chest pain 22 hours after undergoing revascularization of his left anterior descending artery due to acute MI. The patient complains of severe chest pain that is worse upon inspiration and is relieved by sitting up and leaning forward. Echocardiography shows no evidence of tamponade. Which of the following is indicated in the treatment of this patient? a. Administer indomethacin b. Perform pericardiocentesis c. Administer prednisone d. Perform emergent cardiac catheterization e. Administer clopidogrel

a. Administer indomethacin - indicated for acute pericarditis

A 6-year-old boy presents to his PCP with the finding shown (bullseye rash). His father thinks it has been there for about a day. The boy denies other symptoms, although his father remarks that he had a headache last night and seems a bit more fatigued than usual. The family lives in a wooded area, has not traveled at all recently, and has no pets. He has no significant past medical history and no known drug allergies. What is the appropriate treatment? a. Amoxicillin b. Doxycycline c. Azithromycin d. Cephalexin e. Ceftriaxone

a. Amoxicillin - indicated for children under 8

A 60-year-old male presents with palpitations. He reports drinking many glasses of wine over several hours at a family wedding the previous evening. An EKG reveals absent P waves and irregularly irregular rhythm. He does not take any medications. Which is most likely responsible for the patient's symptoms? a. Atrial fibrillation b. Transmural myocardial infarction c. Untreated hypertension d. Torsades de pointes e. Ventricular hypertrophy

a. Atrial fibrillation

A 19-year-old female is brought to a psychiatry clinic by her parents stating that she is "too skinny." On exam, you note an extremely thin female, with body hair shown. Which of the following, if found during the history, physical, or subsequent labs, would result in hospitalization for this patient? a. Body weight less than 75% of average b. Amenorrhea for longer than 3 months c. The patient had an intense fear of gaining weight d. The patient was of the purging subtype e. The patient was found to be anemic

a. Body weight less than 75% of average

A 44-year-old male factory worker presents with a 7-month history of pain and paresthesias involving the volar aspect of the right thumb, index finger, long finger, and the radial half of the ring finger. He has a history of anemia and obstructive sleep apnea. Percussion over the volar wrist crease produces electric sensation distally in the hand and wrist flexion with the elbow in extension produces thumb paresthesias. What is most likely diagnosis? a. Carpal Tunnel Syndrome b. Cubital Tunnel Syndrome c. Cervical Myelopathy d. Ulnar Tunnel Syndrome e. Multiple Sclerosis

a. Carpal Tunnel Syndrome

A 68-year-old woman presents to the ED complaining of ear pain and purulent discharge. On otologic examination, the tympanic membrane is found to be intact with erythematous and edema of the external canal. Which of the following could predispose this patient to this condition? a. Diabetes mellitus b. Hypertension c. Hypercholesterolemia d. Neurofibromatosis Type II e. Frequent swimming

a. Diabetes mellitus

A 52-year-old woman presents to the ED with abdominal pain that began acutely a few hours ago. She additionally complains of nausea and vomiting. Laboratory results show serum amylase of 415 U/L and serum lipase of 520 U/L. The patient reports a history of HIV, diabetes, asthma, anxiety, and glaucoma, all of which are well-controlled with medication. A medication used to treat which of this patient's diseases is most likely responsible for his current presentation? a. HIV b. Diabetes c. Asthma d. Anxiety e. Glaucoma

a. HIV - Didanosine, an anti-retroviral used in HIV, is by far the most common cause of drug-induced pancreatitis

A 45-year-old male undergoes a parathyroidectomy following the passage of two renal stones in a 12-month period. A single parathyroid adenoma is removed. Two days following surgery, the patient reports numbness and tingling around his mouth and lips and muscle cramps. What is the most appropriate next step in the management of this patient? a. IV calcium gluconate b. Vitamin D supplementation c. IV potassium phosphate d. Surgical neck re-exploration e. Observation

a. IV calcium gluconate - this is post-op hypocalmemia

A 21-year-old male presents with diffuse pain. Specifically, he complains of back, left knee, and left heel pain of a week duration. His only significant medical history is that he was treated for penile discharge three weeks ago at his student health clinic. His temp is 98.0 degrees F, BP is 115/80 mmHg, pulse is 82/min and regular, and respirations are 12/min. On exam you notice mouth ulcers. Synovial fluid analysis shows many polymorphonuclear leukocytes, a negative gram stain, and a white blood cell count of 10,000/mm3. What is the appropriate treatment for this patient? a. Nonsteroidal anti-inflammatory drugs (NSAIDs) b. Methotrexate c. Antibiotics d. Colchicine e. Allopurinol

a. Nonsteroidal anti-inflammatory drugs (NSAIDs) - treatment of choice for reactive arthritis

A 40-year-old female is brought to the ED by her husband complaining of severe tightness in her chest. She said that she began sweating, breathing heavily, and felt an impending sense of doom shortly following an argument with a daughter while preparing dinner. She denies any drug use. Vital signs are stable and serum troponins are negative. Her husband reports that she has had several of these episodes in the past. Alprazolam is administered. Which of the following is an appropriate long-term treatment for this patient? a. Paroxetine b. Buspirone c. Metoprolol d. Aspirin e. Phenelzine

a. Paroxetine - SSRIs are used in long term management

Which of the following represents an appropriate pharmacologic treatment algorithm for generic major depression in a 30-year-old male with no comorbidities? (starting at left as first line therapy, and working to right)? a. Paroxetine -> Citalopram -> Amitriptyline b. Sertraline -> Amitriptyline -> Phenelzine c. Fluoxetine -> Amitriptyline -> Nortriptyline d. Phenelzine -> Paroxetine -> Amitriptyline e. Nortriptyline -> Citalopram -> Phenelzine

a. Paroxetine -> Citalopram -> Amitriptyline

A 27-year-old woman presents to clinic in distress. She has suffered from acne all of her life and nothing has seemed to work. Her past treatments include benzoyl peroxide, topical tretinoin, and oral and topical antibiotics. She has no significant past medical history. Her PE is within normal limits with the exception of prominent scarring and nodulocystic acne. You begin her on the most appropriate treatment for this clinical scenario. Regarding this medication, which of the following is true? a. Prior to starting the medication, the patient must agree to using two forms of contraception (i.e. barrier contraceptive and OCP) b. The medication is indicated as a first-line approach for comedones c. The medication is associated with development of bipolar disorder d. The medication is associated with agranulocytosis e. Patients should have a pregnancy test after starting the medication

a. Prior to starting the medication, the patient must agree to using two forms of contraception (i.e. barrier contraceptive and OCP)

A 62-year-old otherwise healthy woman who recently retired from her job as a financial planner presents to her PCP complaining of pain in her left shoulder with any movement. She has recently began preparing her garden for the spring and has had a very difficult time lifting even small planters above her head. She reports that she has been using her right arm to support her left arm when carrying items. Once she lifts her left arm up a bit with the help of her right, she is able to continue lifting the left arm without additional assistance. Where is the most likely site of injury? a. Supraspinatus b. Infraspinatus c. Teres minor d. Subscapularis e. Latissimus dorsi

a. Supraspinatus

A 52-year-old man with a history of hypertension and hyperlipidemia presents to his PCP for an annual physical. On careful physical exam, a pulsatile abdominal mass is noted. A contrast CT of the abdomen reveals an abdominal aortic aneurysm. Surgical repair of the aneurysm is indicated if which of the following are present? a. Symptoms b. Smoking history c. Diameter >2 cm d. Slow rate of growth e. Marfan's syndrome

a. Symptoms - suffering from aneurysm

A 29-year-old female bank teller was working when her bank was robbed. She presents to the psychiatry clinic with anxiety and trouble going to work following the incident. Which of the following aspects of her history would favor the diagnosis of acute stress disorder over post-traumatic stress disorder? a. The event occurred 3 weeks ago b. The event occurred 9 weeks ago c. She experienced feelings of dissociation d. She expresses a feeling of helplessness e. She describes difficulty sleeping

a. The event occurred 3 weeks ago

A 23-year-old G1P0 female at 11 weeks gestation presents to her obstetrician's office for a prenatal visit. PMH is unremarkable. The patient's medications include prenatal vitamins. She does not smoke and has not used alcohol since learning of her pregnancy. Ultrasonography shows an empty gestational sac. PE demonstrates a cervix that is long, closed, and posterior. Which of the following is the most appropriate next step in the management of this patient? a. Emergency dilation and curettage b. Elective dilation and curettage c. Serial beta HCG titers d. Broad spectrum antibiotics e. Broad spectrum antibiotics followed by dilation and curettage

b. Elective dilation and curettage - presents with empty gestational sac without vaginal bleeding or cervical dilation consistent with missed abortion

A 28-year-old female medical student returns from her summer trip to Russia. She is in good health and has no medical illnesses. She follows up regularly with her PCP and is up to date on all of her healthcare maintenance. In preparation for beginning her third year of medical school she receives a PPD which reveals the following as shown with 15mm of induration. Her prior PPDs have all been negative. She has a subsequent QuantiFERON test which is positive. A chest x-ray is clear. She begins isoniazid. Two weeks later, liver function tests reveal AST and ALT of 69 and 79 respectively. Regarding this patient, what is the appropriate next step in management? a. Continue to monitor LFTs and continue isoniazid treatment for 6 months b. Continue to monitor LFTs and continue isoniazid treatment for 9 months c. Continue to monitor LFTs and continue one half the normal dosage of isoniazid treatment for 6 months d. Continue to monitor LFTs and continue one half the normal dosage of isoniazid treatment for 9 months e. Discontinue isoniazid and any further treatment

b. Continue to monitor LFTs and continue isoniazid treatment for 9 months - mild elevation in ALT and AST can be seen in the first few weeks of treatment

A 28-year-old woman presents to the urgent care complaining of weakness, confusion, and that her heart is "racing and flopping" in her chest. She has no significant PMH. She denies any previous episodes of anxiety attacks or heart issues. She smokes 1/2 of a pack of cigarettes per day and is a social drinker. She denies the use of any illicit drugs. She has no known drug allergies, and she does not take any medications on a daily basis. An EKG is ordered. This patient's ventricular contraction rate is determined by which of the following? a. SA node refractory period b. AV node refractory period c. Bundle of His conduction speed d. Purkinje fiber conduction speed e. Purkinje fiber refractory period

b. AV node refractory period - in atrial fibrillation, the ventricular rate is determined by the AV node refractory period

A 48-year-old gentleman presents to his primary care physician complaining of epigastric pain and excessive belching. He states that he has recently been having black, tarry stools. An upper endoscopy is performed and reveals a gastric ulcer. How would food most likely affect this patient? a. All food would make his pain better b. All food would make his pain worse c. All food will have no effect on his pain d. Acidic foods would improve his pain e. Basic foods would improve his pain

b. All food would make his pain worse - this is a gastric ulcer

A 39-year-old woman presents to clinic with multiple complaints. She has trouble swallowing both solids and liquids. She also reports that when she goes out in the cold, her fingers get extremely pale. Vital signs are stable. PE reveals calcinosis of the digits and sclerodactyly as well as telangiectasia over the digits and at the nail bed. One would expect which of the following diagnostic tests to be positive? a. Antihistone antibodies b. Anticentromere antibodies c. Antidesmoglein antibodies d. Antimitochondrial antibodies e. Anti-glomerular basement antibodies

b. Anticentromere antibodies

A 37-year-old man presents to general medical clinic with dysphagia. He notes that his symptoms began several weeks ago and have worsened over time. He now has trouble swallowing solids and liquids, though liquids have always given him the most trouble. He denies any other symptoms. He has no significant PMH. Travel history reveals a recent trip to South America. Vital signs are stable. PE is within normal limits. He has no palpable masses. What is the next step in management? a. Upper endoscopy b. Barium esophagram c. Esophageal manometry d. CT of the chest e. Administer nifurtimox

b. Barium esophagram - next step in management and should precede endoscopy in patients with dysphagia and a broad differential diagnosis

A 32-year-old-man presents to the ED with subacute worsening of chronic chest pain and shortness of breath. Vital signs areas follows: T 37C, HR 81 bpm, NP 127/83 mm Hg, RR 21/min, and O2 saturation of 99% on room air. Cardiac auscultation reveals a crescendo-decrescendo murmur, loudest under the right clavicle, peaking in late systole. Cardiac exam is also notable for a diminished A2. Troponins are negative. An EKG shows left axis deviation and meets criteria for left ventricular hypertrophy. An echocardiogram is ordered, and results are pending. The most likely cause of this man's symptoms is: a. Senile calcific changes b. Bicuspid aortic valve c. Acute bacterial endocarditis d. Myocardial infarction e. Cardiac myxoma

b. Bicuspid aortic valve - signs and symptoms of HF due to aortic stenosis - most common cause in young patient is bicuspid aortic valve

A 17-year-old girl enters pediatric clinic with her mother for a general health visit. Her mother reports that she has been quite concerned with her recent weight loss. Vital signs are stable with the exception of her body mass index of 15 kg/m2. When the mother is asked to leave the room, the patient reports that she has been having trouble with her body image and that she is concerned because she has recently stopped having her period. The remainder of the PE is within normal limits. A pregnancy test is negative. The patient is diagnosed with anorexia nervosa. Which of the following distinguishes anorexia nervous from bulimia nervosa? a. Parotitis b. Body weight less than 85% of ideal body weight c. Alkalosis d. Dorsal hand calluses e. Use of laxatives and diuretics

b. Body weight less than 85% of ideal body weight

A 45-year-old male presents to the ED following a seizure. The patient suffered from an upper respiratory infection complicated by sinusitis 2 weeks ago. The patient's PMH is remarkable for hypertension for which he takes hydrochlorathiazide. Temperature is 39.5C, blood pressure is 120/60 mmHg, pulse is 85/min, and respiratory rate is 20/min. Upon interview, the patient appears confused and exhibits photophobia. Blood cultures are obtained. Which of the following is the most appropriate next step in the management of this patient? a. Ceftriaxone b. Ceftriaxone and vancomycin c. Ceftriaxone, vancomycin and ampicillin d. MRI of the head e. Head CT

b. Ceftriaxone and vancomycin

A 17-year-old previously healthy, athletic male suddenly falls unconscious while playing soccer. His athletic trainer comes to his aid and notes that he is pulseless. He begins performing CPR and on the patient until the ambulance arrives and pronounces the teenager dead. What is the mechanism behind the most likely cause of death? a. Myocardial infarction b. Heart failure due to diastolic dysfunction c. Arrhythmia d. Outflow obstruction e. Rapid volume loss

c. Arrhythmia - this is hypertrophic cardiomyopathy

A 31-year-old male is found down and unresponsive in an alley by the police and brought to the local Level 1 Trauma Center. The event was unwitnessed. On arrival, his Glasgow Coma Score is 3. His vital signs are T 34.2, HR 130, BP 75/44, RR 8, and O2 79% on room air. His physical exam is notable for bilateral periorbital and retroauricular ecchymoses as well as multiple stab wounds to the anterior chest. There are no audible breath sounds on the left chest. Within 5 minutes of arrival, pulses are no longer palpable and the patient is found to be in asystole. Which of the following is the most appropriate first step in the initial management of this patient? a. Chest compressions b. Endotracheal intubation c. Fluid resuscitation d. Central line placement e. Cardioversion

b. Endotracheal intubation - assess and establish airways Follow ABCs

A 27-year-old woman presents to her gynecologist for follow-up for irregular, painful menstrual cycles. Her work-up is negative. She is very distressed about her condition and states that she thinks it's probably related to whatever is causing her bouts of generalized "weakness," which have been occurring about once a month for the last year and associated with headache. She has a PMH of several food intolerances and IBS, both of which cause abdominal pain and managed by a gastroenterologist. She states that she's recently had episodes of chest pain, lasting 20-30 minutes, unrelated to exercise, and requests to be referred to a cardiologist and a neurologist. On exam, she is a thin, athletic woman who appears healthy. What is the most appropriate next step in treatment? a. Discharge her from care b. Establish a primary care provider c. Send her directly to the ER d. Refer her to a psychiatrist e. Refer her to both a cardiologist and a neurologist

b. Establish a primary care provider - she has somatization disorder. One of the most important steps in treatment for this disorder is regular (at least monthly) follow-up with a consistent primary physician the patient trusts.

An 81-year-old year old male presents to his PCP for an annual checkup. He generally feels well except being "old." He has no significant PMH except for benign prostatic hyperplasia, hip replacement, and cataract surgery. On PE, he has a temperature of 37.1C, HR of 87 bpm, and a BP of 151/70. He returns to his physician 1 month later with a pressure of 149/71. Which of the following is the most appropriate management for this patient? a. Explain he has isolated systolic HTN seen in most elderly patients and requires no treatment b. Explain that he has isolated systolic HTN and recommend starting a thiazide c. Explain that he has isolated systolic HTN and recommend starting a beta-blocker d. Explain that he has isolated systolic HTN and recommend starting a fast-acting calcium channel blocker e. Explain that he has isolated systolic HTN, which can increase the risk of stroke and heart disease, but the side effects of the drugs would be of greater concern to this patient than the natural disease course

b. Explain that he has isolated systolic HTN and recommend starting a thiazide

A 23-year-old male presents to the ED after a fall from a ladder. He states that he did not strike his head or suffer loss of consciousness, but that he "hurts all over." He is hemodynamically stable. On secondary survey, you note that he is maintaining his left shoulder in adduction and internal rotation, with ninety degrees of flexion at the left elbow. He is tender to palpation over the left clavicle, but there is no tenting of the skin. Pulses and sensation are intact in the left upper extremity. Appropriate imaging is seen. What is the most appropriate next step in management? a. Weight-bearing as tolerated b. Figure-of-eight immobilization device c. Coaptation splinting d. Open reduction and internal fixation e. Closed reduction with external fixation

b. Figure-of-eight immobilization device - this patient has a closed, minimally displaced fracture of the middle third of his left clavicle

A 65-year-old man with a history of ischemic cardiomyopathy, congestive heart failure, diabetes mellitus, and chronic kidney disease presents to the emergency room with progressive dyspnea on exertion and weight gain for 8 days. Vitals signs are T 99.0, HR 110, BP 130/90, RR 24, SpO2 94% on room air. PE reveals an S3 gallop, 2+ peripheral pitting edema, and marked jugular venous distention. Laboratory results show a serum creatinine of 1.2 mg/dL compared with the patient's normal baseline value of 1.1 mg/dL. A serum troponin is drawn and found to be 0.04 ng/mL. Which of the following medications is indicated first in the care of this patient? a. Enalapril b. Furosemide c. Dobutamine d. Hydralazine e. Dopamine

b. Furosemide - patient is volume overloaded

A 75-year-old female with a long history of debilitating osteoarthritis presents to clinic complaining of gnawing abdominal pain. She has been unable to eat anything substantial for days, as food seems to aggravate the pain. Review of systems is positive for darker stools. Vital signs are stable. Abdominal exam is notable for tenderness to palpation along and just below the epigastrium. What is the most likely diagnosis? a. Duodenal ulcer secondary to H. pylori b. Gastric ulcer secondary to chronic NSAID use c. Lower gastrointestinal bleed d. Squamous cell carcinoma of the esophagus e. Perforated peptic ulcer

b. Gastric ulcer secondary to chronic NSAID use

A 29-year-old man presents to clinic for a general health exam accompanied by his girlfriend. In the office, he seems to be running from topic to topic without a clear message. The patient's girlfriend reports that he took steroids recently for a sinus infection and since then his behavior has been abnormal. He has not had a normal night of sleep for the past ten days, and he just bought a new sports car that he could not afford. She has also caught him with multiple other women in the past few days. The PE is benign and the patient's vital signs are within normal limits. Regarding this patient, which of the following is true? a. His behavior is consistent with a manic episode but does not meet criteria for bipolar I disorder b. His behavior is consistent with a manic episode and also meets criteria for bipolar I disorder c. His behavior is consistent with a manic episode and also meets criteria for bipolar II disorder d. His behavior is consistent with a major depressive episode e. His behavior is consistent with dysthymia

b. His behavior is consistent with a manic episode and also meets criteria for bipolar I disorder - one manic episode is enough for a diagnosis of bipolar I

A 42-year-old male is visiting his physician for symptoms of the flu. On social history, the patient describes drinking 2-3 beers per day as well as smoking 1 pack of cigarettes per day. He is noted on PE to have an oral lesion similar to that shown, which could not be scrapped off with a tongue depressor. This is an example of which of the following? a. Fungal infection b. Hyperplasia c. Neoplasia d. Metaplasia e. Trauma

b. Hyperplasia

A 30-year-old woman is brought to the ED following a MVA. She has no significant PMH and takes no medications. Her temp is 37.5 C (99.5 F), BP is 70/40 mm Hg, heart rate is 120/min, and RR is 22/min. On PE, she is in acute distress due to pain. She is alert and oriented X 3 with no signs of head trauma. She has chest wall bruising bilaterally with tenderness to palpation. Cardiac and pulmonary auscultation are WNL. Her abdominal exam reveals tenderness to palpation with rebound. There is also severe pain with palpation and passive movement of the right hip. Given this patient's hemodynamic instability, she is taken to the operating room, where exploratory laparotomy reveals a splenic laceration and significant hemoperitoneum. She receives twelve units of packed red blood cells during the procedure. On the first post-operative day, her PE reveals hyperactive DTR. Which of the following electrolyte abnormalities is most likely present? a. Hypercalcemia b. Hypocalcemia c. Hyperkalemia d. Hypokalemia e. Hypermagnesemia

b. Hypocalcemia - caused by blood transfusion

A 46-year-old male was found unconscious in the field and brought to the ED by EMS. The patient was intubated in transit and given a 2 liter bolus of normal saline. On arrival, the patient's blood pressure is 80/60 mmHg and temperature is 37.5C. Jugular veins are flat and capillary refill time is 4 seconds. Vascular parameters are measured and are as follows: Cardiac index - Low; Pulmonary capillary wedge pressure (PCWP) - Low; Systemic vascular resistance - High. Which of the following is the most likely diagnosis? a. Septic shock b. Hypovolemic shock c. Anaphylactic shock d. Neurogenic shock e. Cardiogenic shock

b. Hypovolemic shock

A 69-year-old male presents with severe LLQ pain. Vitals are remarkable for a temperature of 38.3 C. Labs are remarkable for a WBC count of 15,000. An abdominal CT scan shows a localized abscess of 2.5 cm near an inflamed diverticulum. What is the next step in the management of this patient? a. Oral antibiotics b. IV antibiotics c. CT-guided percutaneous drainage d. Laparotomy e. NPO, IV fluids, and observation

b. IV antibiotics - recommended initial treatment for abscesses < 3 cm

A 20-year-old G0 female presents with 4 days of fever, pelvic pain, and left-sided adnexal tenderness. PE reveals cervical motion tenderness and left adnexal fullness. ESR is elevated. CBC values are as follows: White blood cell count: 17,000/mm^3 Hemoglobin: 12 g/dL Hematocrit: 36% Platelet count: 200,000/mm^3. Which of the following is the most appropriate treatment for this patient's condition? a. Surgical evacuation b. IV cefoxitin and IV doxycycline c. Oral levofloxacin and oral metronidazole d. IV fluids and bedrest e. Oral azithromycin and oral ceftriaxone

b. IV cefoxitin and IV doxycycline - this is severe disease

A 6-year-old Caucasian boy presents to the pediatrician for his annual physical. The patient's mother states that he has been limping for the past 2 months and that he has occasionally complained of mild discomfort in his right thigh and knee. The mother denies that the patient has had recent trauma or fever. The patient's vital signs are within normal limits. On exam, there is no edema in the patient's right knee but there is limited internal rotation and abduction of the right hip. The pediatrician also notes that the patient has an asymmetric gait that results in less time bearing weight on his right side. The plain radiograph of his pelvis is demonstrated. CBC is within normal limits. Which of the following is the most likely diagnosis? a. Septic arthritis b. Legg-Calve-Perthes disease c. Osgood-Schlatter disease d. Osteomyelitis e. Slipped capital femoral epiphysis

b. Legg-Calve-Perthes disease

A 56-year-old woman is in the ER after a seizure. Although suffering from some continued confusion, she keeps pointing at the area around her mouth and saying that it feels "funny." Her husband states that she has a history of osteoarthritis and thyroid cancer, for which she underwent a total thyroidectomy one month ago. As far as he knows, she has no family history of epilepsy. Her exam is notable for 3+ reflexes in her upper and lower extremities. After applying a blood pressure cuff to her arm for 3 minutes, her hand looks like this. Which laboratory abnormality is her blood work most likely to reveal? a. Low phosphorous b. Low ionized calcium c. Low magnesium d. High 25-hydroxyvitamin D e. High PTH

b. Low ionized calcium - post thyroidectomy

A 29-year-old G2P1 at 40 weeks gestation presents to the hospital with regular uterine contractions at 10 minute intervals that have been steadily increasing in strength. PE reveals a cervix dilated to 3 cm with 60% effacement. You tell the patient that she is in labor and estimate that her cervix is dilating at a rate of 1.5 cm per hour. Which of the following will mark the end of the first stage of labor in this patient? a. Delivery of the placenta b. Maximal cervical dilation c. Delivery of the infant d. Positive fern test e. A fetal head station of +2

b. Maximal cervical dilation

A women who is 20-weeks pregnant presents to your clinic with mild dysuria and increased urinary frequency. She is afebrile. PE is negative for costovertebral angle tenderness. What is the best agent to treat this condition? a. Trimethoprim-sulfamethoxazole b. Nitrofurantoin c. Ciprofloxacin d. Ampicillin e. Penicillin

b. Nitrofurantoin

A G0 57-year-old female presents to her gynecologist complaining of lower abdominal pain, early satiety, and urinary frequency of 2 weeks duration. She used oral contraceptive pills (OCPs) for several years during her twenties and thirties, and entered menopause at age 45. Surgical history is notable for a myomectomy 20 years ago. FH reveals that her mother died of lung cancer. Lab results demonstrate markedly elevated CA-125. A pelvic CT scan is shown. Which of the following in this woman's history is a risk factor for ovarian carcinoma? a. Family history of lung cancer b. Nulliparity c. Oral contraceptive use d. Myomectomy e. Early menopause

b. Nulliparity

A 34-year-old African American female presents to your office complaining of dry cough, dyspnea and blurry vision. She reports her symptoms began two months ago and have progressively worsened. Currently, climbing a single flight of stairs is difficult. She reports dark, floating 'spots' in her visual field and says bright lights make her uncomfortable. PMH is insignificant. The patient's only medications are estrogen/progesterone oral contraceptives. Laboratory values are notable for elevated serum calcium. Chest X-ray is shown. Which of the following is the most appropriate management for this patient? a. Furosemide b. Oral corticosteroids c. Calcitonin d. IV methotrexate e. Observation while waiting for disease remission

b. Oral corticosteroids - progressive dyspnea, symptoms of anterior uveitis, hypercalcemia, and bilateral hilar lymphadenopathy consistent with sarcoidosis - treated with oral corticosteroids

A 45-year-old male with no significant medical history comes to your office complaining of pain and stiffness in his hands upon wakening in the morning. You perform an x-ray. What is the most likely diagnosis? a. Gouty arthritis b. Osteoarthritis (OA) c. Rheumatoid arthritis (RA) d. Psoriatic arthritis (PsA) e. Systemic lupus erythematosus (SLE) HINT: focus on the joints affected

b. Osteoarthritis (OA) - x-ray demonstrates complete loss of the articular cartilage at all four DIP joints, large osteophytes (bone spurs), and ankylosis of the DIP joint of the middle finger

A 21-year-old female college student presents to the clinic complaining of intermittent diarrhea, abdominal pain, and bloating. She has had a feeling of general weakness and fatigue over the last month. She states that her stool has recently seemed to be "grease coated". She does not have any significant PMH. Vital signs are T 98.9 F, HR 103, BP 110/75, RR 16, and SpO2 99%. Laboratory tests show an iron deficiency anemia. On PE, what skin condition is most likely associated with her presentation? a. Red and white scaly patches or plaques on the top layer of the epidermis b. Papulovesicular eruptions, usually distributed symmetrically on extensor surfaces c. Pinkish-red, flaky, oval-shaped large patches on the torso in a characteristic "Christmas tree" distribution d. Redness with yellow-grey-white flakes on the face, behind the ears, and in various skin folds e. Tense bullae of the inner thighs, upper arms, trunk and extremities

b. Papulovesicular eruptions, usually distributed symmetrically on extensor surfaces This is dermatitis herpetiformis, a chronic blistering, pruritic papulovesicular skin condition associated with celiac disease

A 84-year-old female nursing home resident is admitted to the MICU following a severe episode of urosepsis. Several days into a course of appropriate antibiotic therapy, her white blood cell count begins to rise. As part your workup, you perform a right upper quadrant ultrasound which shows a distended gall bladder with a thickened wall and pericholecystic fluid. What is the most appropriate next step in the management of this patient? a. Emergent cholecystectomy b. Percutaneous cholecystostomy c. Abdominal CT scan d. HIDA scan e. Observation

b. Percutaneous cholecystostomy - acalculous cholecystisis is seen in chronically ill patients

A 19-year-old college student presents to the pediatric emergency department complaining of an itchy rash on her hands and in her groin for 2 days. The itching is worse at night. She and her boyfriend recently completed a rabbit taxidermy class; he has similar symptoms. Review of systems, vital signs, and physical exam are otherwise normal except for the rash on the patient's hand as shown in Image A. Which of the following is the most appropriate pharmacologic treatment? a. Lindane b. Permethrin c. Streptomycin d. Nystatin e. Praziquantel

b. Permethrin - first line

A 28-year-old swimmer presents to an outpatient clinic with ear pain especially with traction of the external ear. Otoscopic exam reveals an inflamed ear canal with cerumen present. A diagnosis of otitis externa is made. Before topical antibiotics are prescribed, which of the following must occur? a. Culture of any purulent discharge b. Removal of cerumen with wire loop c. Irrigation of the ear with hydrogen peroxide d. Analgesia of the ear e. Start IV antibiotics

b. Removal of cerumen with wire loop

The counselor at your town's high school calls to ask your advice about a student. The counselor is currently meeting with all juniors to discuss plans after graduation. She tells you one of the students she spoke with today exhibited some concerning behaviors and wonders if she should suggest that the parents bring her to a psychiatrist. Reportedly, the student is a "loner" and seems to express no emotion. He states that he has no friends and prefers spending time at home. He denied any depressive symptoms and his thinking seemed logical, though strange. Which of the following best fits this student? a. Acute psychosis b. Schizoid personality disorder c. Schizotypal personality disorder d. Schizoaffective disorder e. Paranoid personality disorder

b. Schizoid personality disorder - pattern of social withdrawal, isolation, lack of interest in forming relationships, restricted emotional expression and lack of distorted cognition or reason

An 18-month-old child is brought in by her mother for symptoms of irritability and poor oral intake. On otoscopic exam, the tympanic membrane is noted to be bulging and hyperemic, with no light reflex. Which of the following organisms is the most likely the cause of this child's acute condition? a. Haemophilus influenze b. Streptococcus pneumoniae c. Moracella catarrhalis d. Rhinovirus e. Respiratory syncitial virus

b. Streptococcus pneumoniae

A 27-year-old woman presents with pain during sexual intercourse. She also reports severe menstrual-related pain. Around the time of her periods, she complains of pain with bowel movements or urination. She has no significant PMH. Vital signs are stable. PE reveals no palpable masses and a pelvic exam which is WNL. Which of the following is true about this woman's condition? a. This condition is associated with insulin resistance b. The patient is at a greater risk of infertility c. The patient is at a greater risk of ovarian or breast cancer d. This patient should be referred for a hysterectomy e. Having multiple children is a risk factor for this illness

b. The patient is at a greater risk of infertility - this is endometriosis

A 45-year-old man with no known past medical history is presenting to your primary care office for the first time after discovering that he had a blood pressure of 150/85 at a screening event at his local community center two weeks ago. His only complaint is "heartburn" which occurs after large meals which is relieved by over the counter antacids. His physical exam is unremarkable except for a blood pressure of 160/90. Which of the following tests should be performed at this visit? a. CBC b. Urinalysis c. Chest X-ray d. Colonoscopy e. Esophagogastroduodenoscopy

b. Urinalysis - new diagnosis - needs UA for signs of long term kidney damage

A 29-year-old female presents to clinic for an evaluation of her asthma. She was given a prescription by her previous physician for an albuterol inhaler which she has been using. She reports symptoms on average three days per week. Last month she was awakened by her asthma on three nights. She is able to participate in light jogging but high intensity aerobic exercise often triggers her symptoms. Which of the following is the best next step in management? a. Begin a short course of oral prednisone b. Continue the albuterol inhaler PRN c. Add a long-acting inhaled corticosteroid d. Add a long-acting beta-agonist e. Add a leukotriene antagonist

c. Add a long-acting inhaled corticosteroid

A 55-year-old man recently status post abdominal surgery is discharged from the PACU to the inpatient surgery ward where he remains NPO with an NG in place to suction for bowel rest. You are called to the bedside because his blood pressure is 182/89. His heart rate is 89. He complains of 3/10 pain at the surgical site only. Recent lab results were within normal limits. You learn that he stopped taking his blood pressure medications (metoprolol 25mg twice daily) one week before surgery. Of the following choices, which is the best option for managing this patient's blood pressure? a. Resume 25mg oral metoprolol twice daily b. Increase to 50mg oral metoprolol twice daily c. Administer 5mg metoprolol via IV d. Administer 1 mg/kg esmolol via IV e. Better control the patient's pain

c. Administer 5mg metoprolol via IV - consistent with hypertensive urgency

A 23 year-old woman is brought to the ED by her mother after she is found to have cut both of her wrists with razor blades. The patient admits a history of self-mutilation and attributed this incident to a recent breakup with a man she had been seeing for the previous two weeks. On morning rounds the patient reports to the team that "the nurses here are the worst I've ever seen, but all the doctors are absolute geniuses." After her immediate injuries are addressed, which of the following should be used as a first-line therapy for the patient's psychiatric condition? a. Alprazolam b. Amitriptyline c. Behavior therapy d. Fluvoxamine e. Classical conditioning

c. Behavior therapy - this is borderline personality disorder

A 21-year-old female college student presents with headache, malaise, and fever of 3 hours duration. PMH is unremarkable and the patient does not take medications. She denies recent alcohol or drug use. Temperature is 39°C, blood pressure is 110/70 mmHg, pulse is 76/min and respirations are 17/min. Physical examination is notable for a petechial rash present in the lower extremities bilaterally. Extension of the knee when both the knee and thigh are flexed produces pain. Which of the following is the most appropriate next step in management? a. MRI of the head b. Ampicillin c. Blood cultures d. Head CT e. Ceftriaxone

c. Blood cultures - should be obtained prior to initiation of antimicrobial therapy

A 27-year-old hiker from Massachusetts presented to her PCP with a new rash pictured. She denied any fevers, chills, arthralgias, or other symptoms. She is prescribed the first-line medication for this condition. The patient then returns 1 week later following another hiking trip with erythema, edema, and vesicles on her face, shoulders, and legs. Which of the following could have prevented this from happening? a. Assessing the patient's allergy history b. Testing the patient's serum ANA level c. Cautioning the patient on common side effects of the medication d. Recommending avoidance of wooded areas e. This could not have been prevented

c. Cautioning the patient on common side effects of the medication -Doxycycline can predispose to phototoxicity, patients should be educated on reducing sun exposure

A young Caucasian couple presents to your clinic for an infertility evaluation. The woman is age 26 and the man is age 28. They have been trying to conceive through unprotected sex for the past 15 months. The woman reports irregular menses. PE is notable acne and hirsutism. A semen analysis is within normal limits. Which of the following is most appropriate agent for the initial management of this couple's infertility? a. In vitro fertilization b. FSH c. Clomiphene citrate d. Danazol e. Glyburide

c. Clomiphene citrate - first line treatment for PCOS

A patient with COPD is admitted to the hospital with cholecystitis. He undergoes cholecystectomy uneventfully and is transferred to the floor. Suddenly, he becomes weak and severely hypotensive. If the patient's symptoms are due to adrenal insufficiency from chronic steroid use, one would expect which of the following: a. Elevated renin b. Elevated cortisol c. Decreased ACTH d. Elevated ACTH e. Elevate aldosterone

c. Decreased ACTH - this is secondary adrenal insufficiency

A 3-year-old male presents to the ED with difficulty breathing. On PE, the patient is febrile and tachypneic. The patient's lateral neck radiograph is shown. Which of the following signs on physical examination is most specific for this patient's disease? a. Cough b. Stridor c. Drooling d. Rhinorrhea e. Conjunctivitis

c. Drooling - most specific for epiglottis

A concerned mother brings her previously healthy 5 year child to the family care physician after 3 mornings of significant swelling around the child's eyes and feet. The mother is concerned since she feels that the child has not been urinating at his normal frequency, and that his urine has been "thick and frothy." The physician requests a 24hr urine protein collection which reveals nephrotic range proteinuria. What is the next step in the management of this patient? a. Echocardiogram b. Abdominal X-Ray c. Empiric Steroid Therapy d. Renal Biopsy e. Renal Ultrasound

c. Empiric Steroid Therapy - this is minimal change disease

A college tennis player complains to her coach that she has constant shooting pain along the lateral aspect of the elbow that radiates down her forearm. Repeated forceful use of which muscle or muscle group is the most likely etiology of her symptoms? a. Deltoid b. Triceps c. Extensor carpi radialis d. Extensor carpi ulnaris e. Flexor carpi ulnaris

c. Extensor carpi radialis - this is tennis elbow, or lateral epicondylitis

You are in the ED seeing a 3-year-old girl who presents with elbow pain. Approximately one hour prior to presentation, the patient was walking with her mother, when she started to run across the street toward her friends who were playing in the yard. Noticing a car coming, the patient's mother quickly grabbed the patient's left arm and pulled her back sharply. The patient immediately began to cry, and is currently holding her arm across her stomach with her palm facing down. Radiographs of her elbow are unremarkable. What is the most appropriate next step? a. Immediate casting b. Urgent surgery c. Flexion and supination of the forearm d. Extension and supination of the forearm e. Expectant management

c. Flexion and supination of the forearm - this is the appropriate action for nursemaid's elbow, subluxation of the radial head

A 56-year-old female presents with several episodes in which she felt "dizzy." These were associated with unilateral "ringing" in her ears as well as a decrease in hearing acuity. Which of the following features distinguishes Menière's disease from benign paroxysmal positional vertigo (BPPV)? a. Vertigo b. Facial paralysis c. Hearing loss d. Episodic nature e. Unilateral nature

c. Hearing loss

A 69-year-old with COPD presents to clinic for follow-up. He has had numerous disease exacerbations and has been on high-dose prednisone for the past two years. His vital signs are: T 37C, HR 80, BP 150/90, RR 14, and O2 99% on room air. His physical examination is notable for these findings. What would be the most likely electrolyte abnormality found in this patient? a. Hypokalemia and hyponatremia b. Hyperkalemia and hypernatremia c. Hypokalemia and hypernatremia d. Hyperkalemia and hyponatremia e. Hypercalcemia and hypernatremia

c. Hypokalemia and hypernatremia

A 17-year-old female is brought to a psychiatrist by her parents after being caught forcing herself to vomit in her high school bathroom. This is the second such episode in which the patient has been found vomiting at school. Which of the following features would, if present, help distinguish this patient's disorder from anorexia nervosa? a. Amenorrhea b. Distorted body self-perception c. Anhedonia d. Insomnia e. Normal body weight

e. Normal body weight

A 23-year-old previously healthy female not currently taking any medications presents to the ED with complaints of chills, malaise, and pain in her left lower extremity. She reports feeling "off" for the last 24 hours, but wanted to complete the 7 hour drive home from her recent hiking trip. Vital signs are as follows: temp101.8F (38.8C), HR of 95 bpm, BP 125/70 mmHg, and respiratory rate of 11 resp/min. PE findings are shown in the figure. You also note a large, poorly-healed blister on the left great toe. While waiting for labs to return, the patient spikes a fever to 103F (39.4C) and begins to feel confused. Along with IV fluids, which of the following is the most appropriate treatment? a. IV heparin b. IV penicillin G c. IV cefazolin d. IV caspofungin e. Oral vancomycin

c. IV cefazolin

A 48-year-old man with a history of alcoholism presents to the emergency department with nausea, vomiting, and severe abdominal pain radiating to the back. He says his symptoms began 5 hours ago. A CT is obtained. What is the most appropriate initial treatment in this patient? a. Gastrografin swallow study b. Surgical intervention c. IV fluids and analgesics d. IV antibiotics e. Upper endoscopy

c. IV fluids and analgesics - supportive treatment for acute panceatitis

A 47-year-old male presents with pain and swelling in his great toe for three days duration. He has never had similar symptoms. He his afebrile, but has erythema and warmth over his great toe. A clinical image is shown. Serum laboratory levels show a elevated uric acid level. The joint is aspirated, and crystal analysis is shown. What is the most appropriate first line of treatment. a. Allopurinol (Zyloprim) b. Colchicine (Colcrys) c. Indomethacin (Indocin) d. Cephalexin (Keflex) e. Surgical irrigation and debridement

c. Indomethacin (Indocin)

A 36-year-old male comes to your office with intermittent right knee pain of 3 weeks duration. He remembers a popping sensation while changing direction during a football game 3 weeks ago that produced moderate swelling in the joint. PMH is unremarkable. On exam, a click is palpated when the examiner places a hand on the medial knee while externally rotating the knee during extension from the flexed position. The patient is unable to fully straighten the right knee. Radiographs of the knee are normal. Which of the following is the most likely diagnosis? a. Anterior cruciate ligament tear b. Posterior cruciate ligament tear c. Medial meniscus tear d. Medial collateral ligament tear e. Lateral collateral ligament tear

c. Medial meniscus tear

A 42-year-old male presents to the ED complaining of 2 days of depressed mood, muscle aches and spasms, and diarrhea. The patient reports a long history of IV drug use and says he drinks half a pint of vodka daily. Three days ago he had his wallet stolen and has since not been able to buy drugs or alcohol. PE is notable for diaphoresis and dilated pupils. Temp is 37.8 degrees Celsius. Which of the following is the most appropriate treatment for this patient? a. Lorazepam b. Alprazolam c. Methadone d. Naloxone e. Flumazenil

c. Methadone - the patient suffers from symptoms of opioid withdrawal

A 24-year-old male presents to the clinic complaining of urethral discharge and pain on urination. On exam, watery, non-purulent discharge is noted at the urethra without evidence of sores or ulcers on the penis. The patient is given a single oral dose of azithromycin and sent home. He returns in 2 weeks with the same complaints. He denies any sexual activity since his initial visit. Which of the following is the most appropriate next step in treatment? a. Longer course of azithromycin b. Ceftriaxone c. Metronidazole d. Sexual health counseling e. Reassurance

c. Metronidazole - consistent with nongonococcal urethritis refractory to azithromycin treatment. Trichomonas vaginalis infection should be suspected.

A 63-year-old male presents with several episodes of feeling dizzy. These episodes were associated with a ringing in his left ear as well as what he perceived as a decrease in hearing acuity. He also reports a sensation of increased pressure on the affected side. Past medical history is significant for HTN for which he takes lisinopril and hyperlipidemia for which he takes atorvastatin. Which of the following is the most likely diagnosis in this patient? a. CN VIII schwanomma b. Brainstem mass c. Ménière's disease d. Benign paroxysmal peripheral vertigo (BPPV) e. Drug-induced ototoxicity 25% (2/8)

c. Ménière's disease - classic presentation - episodic vertigo with hearing loss and aural fullness

You are a surgeon serving in the military overseas caring for patients on the front line. Suddenly you get a new admission transferred by helicopter. The patient is a 25-year-old male. He has been shot in the right lower chest and has been stable for the past two hours. He has been getting progressively more short of breath. Vitals reveal temp 37 degrees Celsius, HR of 121, BP of 100/70, respiratory rate 23, and oxygen saturation of 89% on room air. You hear diminished breath sounds on the right side. A bedside x-ray is taken and reveals the following. What is the next step in management? a. Insert a chest tube b. Place the patient on oxygen and insert a chest tube c. Place a needle thoracotomy d. Obtain an arterial blood gas e. Observation

c. Place a needle thoracotomy - tension pneumothorax after a traumatic gun shot wound - next intervention is needle thoracotomy

A 28 year-old woman G2P1 at 29 weeks gestation presents to the obstetrician after noticing red spots on her undergarments over the past week. Her previous child was born by Cesearean section. She denies any pain associated with this spotting and any contractions. She endorses feeling fetal movements. She has received routine prenatal care up to this point and her pregnancy thus far has been uneventful. What is the diagnosis that must be ruled out prior to the patient being examined? a. Uterine rupture b. Threatened abortion c. Placenta previa d. Placenta abruptio e. Placenta Accreta

c. Placenta previa

A 16-year-old Caucasian boy presents to your family practice office complaining of itchiness. He denies other symptoms. He also denies tobacco, alcohol, or other illicit drug use and is not sexually active. He has no other significant past medical or surgical history aside from a meniscal repair from a wrestling injury sustained two years ago from which he has recovered fully. Vitals are T 98.3, HR 67, BP 110/70. On exam you note several pruritic, erythematous, slightly raised annular patches with central clearing on his back. Which of the following additional tests or features are sufficient to make the diagnosis of this boy's skin lesion? a. Acid-fast bacilli on smear from skin scrapings b. History of recent herald patch and lesions along skin cleavage lines c. Presence of hyphae when KOH added to skin scrapings d. Symmetrical distribution on bilaterial extremities progressing proximally e. History of time spent in a Lyme-endemic region

c. Presence of hyphae when KOH added to skin scrapings - central clearing - tinea corporis

A 39-year-old man presents to the ED with fever and cough. He has no prior smoking history. He is originally from Russia and since immigrating 5 years ago, he has contracted HIV. On the second day of admission, he develops severe hypotension, abdominal pain and acute renal failure. A CT of the abdomen and pelvis reveals extensive calcification of the adrenal glands bilaterally. What is the most likely diagnosis? a. Primary adrenal insufficiency secondary to autoimmunity b. Primary adrenal insufficiency secondary to metastatic lung cancer c. Primary adrenal insufficiency secondary to M. tuberculosis d. Secondary adrenal insufficiency e. Tertiary adrenal insufficiency

c. Primary adrenal insufficiency secondary to M. tuberculosis

A 64-year-old male presents to the ED complaining of "ear pain with fluid coming out." Past medical history is positive for a "sugar problem." The otoscopic exam is shown. A bacterial swab of purulent material in the canal sent for culture would most likely grow which of the following organism? a. Staphylococcus aureus b. Streptococcus pyogenes c. Pseudomonas aeruginosa d. Haemophilus influenzae e. Moraxella catarrhalis

c. Pseudomonas aeruginosa

A 21-year-old college student presents to the student health clinic complaining of a painful rash on the border of her lip. She says that it has come and gone a few times before, and that she usually can feel it before a break out of the lesions. At high doses, the first-line medication to treat this condition can be especially toxic to which organ system? a. Nervous b. Cardiovascular c. Renal d. Pulmonary e. Hepatic

c. Renal - acyclovir can be nephrotoxic at high doses, especially in patients with volume depletion and baseline renal insufficiency

A 62-year-old female presents to general medical clinic for health maintenance. She is due for a colonoscopy but before she schedules it, she would like to have a full exam. She has no complaints and no significant past medical history. She has been in good health for most of her life. Vital signs are stable. Her PE is benign. Routine labs reveal a calcium of 11.2 mg/dL. What is the next step in management? a. Order PTH b. Order PTH related peptide c. Reorder serum calcium d. Order ACE e. Order a chest X-ray

c. Reorder serum calcium - first step

A 45-year-old male with no significant medical history comes to your office complaining of pain and stiffness in his hands upon wakening in the morning for the past several months. The stiffness generally lasts one to two hours, per the patient. You perform an x-ray. What is the most likely diagnosis? a. Gouty arthritis b. Osteoarthritis (OA) c. Rheumatoid arthritis (RA) d. Psoriatic arthritis (PsA) e. Systemic lupus erythematosus (SLE) HINT: focus on the joints affected

c. Rheumatoid arthritis (RA) - involves the MCPs and carpal bones, significant MCP joint damage resulting in ulnar subluxation

A 34-year-old woman with no significant past medical history presents to a cardiology clinic with complaints that her "heart is racing." She says these events have occurred once a week for the past three weeks and do not seem to be initiated by any specific trigger. During these episodes, the patient is short of breath and feels like she is "doomed". A medical workup is undertaken and is negative, including an ambulatory cardiac event monitor that showed sinus tachycardia during one of the patient's episodes. This patient could be diagnosed with panic disorder if which of the following were also present? a. She states that these symptoms appear when she is in public places and feels she "can't escape" b. She experiences severe diaphoresis and vertigo c. She has anxiety about having additional attacks d. She experiences disturbed sleep and muscle tension e. She describes these symptoms as starting after she was in a car accident

c. She has anxiety about having additional attacks

A 15-year-old boy presents to the ED with severe lower abdominal pain that awoke him from sleep about 3 hours ago. The pain is sharp and radiates to the left thigh. While in the ED, the patient experiences one episode of vomiting. He denies any fever, dysuria or chills. PE reveals normal vitals with BP 100/60 and temperature of 98.6F. Abdominal examination is relatively benign. Scrotal examination reveals an elevated left testis that is diffusely tender. What is the next step in the management of this patient? a. CT scan of abdomen and pelvis b. Testicular ultrasound c. Surgical exploration d. IV antibiotics e. KUB x-ray

c. Surgical exploration - most likely left testicular torsion - surgical emergency

A 30-month-old female is brought to your office by her mother who is concerned about her daughter's irritability and the bluish tinge of her torso. A quick perusal of her chart reveals a documented 3/6 harsh, systolic ejection murmur heard best at the upper left sternal border. During the interview the mother abruptly interjects to draw your attention to the child who is in mild distress and sitting with her knees drawn to her chest in the corner. Auscultation of the upper left sternal border reveals her murmur has increased to a 4/6. What is the most likely diagnosis? a. Atrial septal defect b. Ventricular septal defect c. Tetralogy of Fallot d. Transposition of the great arteries e. Patent ductus arteriosus

c. Tetralogy of Fallot - squatting in an attempt to relieve distress

A 65-year-old G0 postmenopausal female presents with vaginal bleeding of two days duration. The patient has DM type II and takes metformin. She has a 30-pack year history of smoking, but denies alcohol or illicit drug use. She denies any history of trauma to the pelvic area. PE reveals no abnormalities. Endometrial biopsy and subsequent work-up reveals endometrial carcinoma. Which of the following is the most appropriate treatment for this patient? a. Total abdominal hysterectomy b. Radiation only c. Total abdominal hysterectomy and bilateral salpingo-oopherectomy d. High dose progestins e. Fractional dilation and curettage

c. Total abdominal hysterectomy and bilateral salpingo-oopherectomy - consistent with Stage I endometrial carcinoma

A 32-year-old man presents to his primary care doctor with the concern of fatigue and pain upon defecation. He denies any diarrhea or constipation, but notes that for the last two months he is having pain with passing stool. He notes a similar episode that occurred approximately one year ago. On exam, you note a horizontal anal fissure. Exam of his oral cavity is shown. Which of the following is the most appropriate step in management? a. Reassurance that these lesions are self-limited b. Lateral internal sphincterotomy c. Upper and lower endoscopy d. Topical steroid ointment e. HIV testing

c. Upper and lower endoscopy - combination of an anal fissure in an aberrant location with apthous ulcers in the oral mucosa is very suspicious for Crohns disease

A 60-year-old Caucasian male presents to your office complaining of shortness of breath on exertion. He undergoes an echocardiogram and is found to have an ejection fraction of 35%. Which of the following classes of drugs would improve mortality in this patient? a. Cardiac glycosides b. Calcium channel blockers c. Thiazide diuretics d. ACE inhibitors e. Nitrates

d. ACE inhibitors

A 34-year-old male with a history of Crohn's disease presents with recent onset periumbilical abdominal pain, weight loss, and fever. On PE he is noted to have a palpable abdominal mass in the right lower quadrant. What is the appropriate next step in management? a. Administration of corticosteroids b. Administration of methotrexate or azathioprine c. Abdominal plain film d. Abdominal CT e. Surgical consult

d. Abdominal CT - concerning for an abdominal abscess and CT is the diagnostic test of choice

An overweight, 42-year-old man comes to the physician complaining that his asthma has been worsening over the past few months. He notes that this change coincided with a recent job promotion in which he works late and has very little time to unwind or do anything but go to bed after dinner. He also has a consistently sore throat and hoarseness in the morning. Occasionally, he experiences chest pain but he does not think it is related to exertion. In addition to asthma, his medical history is significant for HTN. He has taken the same medications for years: albuterol inhaler as needed, low-dose fluticasone inhaler daily, and lisinopril. On PE, there are no abnormalities noted. What is the most appropriate next step in management? a. Discontinue lisinopril b. Add salmeterol c. Switch to high-dose fluticasone inhaler d. Add omeprazole e. Add dextromethorphan as needed

d. Add omeprazole

A 65-year-old man presents to the ED with left-sided chest pain. He says the pain occurs at rest and is localized exclusively to the chest without radiation to the extremities. The patient has had chest pain with exertion for the past three years, but over the past year his symptoms have occurred following activities of daily living. The current episode provoked anxiety and led him to seek medical treatment. Past medical history includes HTN that he treated successfully with exercise and diet modifications. While speaking with the ED physician, the patient reports that the pain has resolved. ECG shows LBBB unchanged from a prior ECG 3 years ago. Serum troponin tests are normal. Which of the following is the most appropriate next step in management in this patient? a. Discharge the patient with nitroglycerin b. Discharge the patient with aspirin, atorvastatin, and hydrochlorothiazide c. Discharge the patient with a follow-up appointment to see a cardiologist d. Admit the patient and schedule an adenosine perfusion stress test e. Admit the patient and schedule an exercise ECG

d. Admit the patient and schedule an adenosine perfusion stress test - patient should not be discharged until extend of CAD is evaluated

A 60-year-old man presents with pain and swelling in his great toe of three days duration. He has never had these symptoms before. On PE he is afebrile, and has erythema over the great toe. A clinical image is shown. Which of the following laboratory or imaging results would confirm the diagnosis of acute gout in this patient? a. Elevated serum uric acid level b. Radiographs showing joint space narrowing of the 1st metarsalphalangeal joint and soft tissue radio-densities c. Magnetic-resonance imaging showing increased joint fluid and T2 signal intensity in the metatarsal head d. Arthrocentesis showing intracellular crystals that are thin, needle-shaped, and strongly negatively birefringent e. Arthrocentesis showing intracellular crystals that are rhomboid-shaped and weakly positively birefringent

d. Arthrocentesis showing intracellular crystals that are thin, needle-shaped, and strongly negatively birefringent

A 47-year-old man with a 20-year history of bipolar disorder type I presents to clinic for a general health exam accompanied by his wife. You notice he is easily distracted. The patient's wife reports that he has not had a normal night of sleep for the past several weeks and that she had to cancel his credit cards because he began spending recklessly. When asked about his sleep pattern, the patient gets agitated. The patient's wife reports that there have been no recent changes in his medication. He is on lithium and lisinopril for hypertension. The PE is benign and the patient's vital signs are WNL. Regarding this patient's medication regimen, what is the next step in management? a. Discontinue all medications b. Begin fluoxetine c. Begin valproic acid d. Begin clozapine e. Begin venlafaxine

d. Begin clozapine - the patient is already on a mood stabilizer

A 72-year-old male, former smoker (50 year pack history), with a history of poorly controlled diabetes and HTN five years status post three-vessel CABG and 20 years status post cholecystectomy is brought to your clinic by his son, who states that his father has lost 18 lbs in the past year. The patient states that he no longer feels like eating (dull, crampy pain in his epigastrium when he eats). He denies alcohol use. His home medications include lisinopril, hydrochlorothiazide, metformin, and aspirin. Chest and abdominal non-contrast CT show no abnormalities, and upper endoscopy demonstrates mild gastritis. What is the most likely diagnosis? a. Colonic malignancy b. Chronic pancreatitis c. Peptic ulcer disease d. Chronic mesenteric ischemia e. Abdominal aortic aneurysm

d. Chronic mesenteric ischemia

A 78-year-old male is transferred from his nursing home to the emergency department because of recurrent coughing spells and regurgitation after meals. There is a note that arrives with the patient stating that his breath has been nearly unbearable during the past few weeks. What is the most appropriate way to diagnose this patient's pathology? a. Clinical observations and history b. Clinical observations, history, upper endoscopy c. Clinical observations, history, chest radiograph d. Clinical observations, history, barium swallow e. Barium swallow

d. Clinical observations, history, barium swallow

A 68-year-old male presents to the emergency department with left lower quadrant pain and fever for 1 day. Laboratory results show: WBC 14.8 cells/mm^3 Hb 12.0 g/dL Hct 38% Na 138 mEq/L K 4.0 mEq/L Creatinine 1.0 mg/dL Which of the following studies is contraindicated in the workup of this patient? a. MRI b. Ultrasound c. Contrast CT d. Colonoscopy e. Abdominal radiograph

d. Colonoscopy - this is acute diverticulitis

A 28-year-old woman who has been breastfeeding her baby for approximately 3 months visits her physician complaining of breast tenderness and fevers. After assessing the patient, the physician believes that she is experiencing mastitis. The patient is started on analgesics and antibiotics. What should the physician recommend in terms of breastfeeding from the affected breast? a. Discontinue breastfeeding to allow time for proper healing b. Discontinue breastfeeding to decrease bacterial spread to the infant c. Discontinue breastfeeding to decrease bacterial spread within the mother d. Continue breastfeeding to decrease the chance of the mastitis progressing to a breast abscess e. Continue breastfeeding since the small bacterial load can help develop the infant's immune system

d. Continue breastfeeding to decrease the chance of the mastitis progressing to a breast abscess

A 28-year-old Caucasian female presents to her primary care physician complaining of a one-week history of fatigue, progressively worsening shortness of breath, and swelling of her feet and ankles. She denies chest pain. Past medical history is unremarkable except for a recent cold two weeks ago. She denies any past surgical history. She takes oral contraceptive pills. She denies any recreational drug use. On physical exam, her temperature is 37 C (98.6 F), blood pressure is 120/70 mmHg, pulse is 84/min, and respiratory rate is 20/min. Her physical exam is also notable for bibasilar crackles, jugular venous distension, an S3 gallop, and 2+ pitting edema up to the ankles bilaterally. Her electrolytes and CBC are within normal limits. Transthoracic echocardiogram of her heart will likely show which of the following? a. Mitral stenosis b. Aortic valve vegetations c. Idiopathic pulmonary arterial hypertension d. Dilated ventricles with diffuse hypokinesia e. Eccentric cardiac hypertrophy

d. Dilated ventricles with diffuse hypokinesia - this is acute HF with dilated cardiomyopathy post viral

A 59-year-old male is brought to the ED after sustaining thoracic trauma following an assault with a baseball bat. He reports shortness of breath. On PE, his vital signs are temperature of 37 C, heart rate 121 bpm, blood pressure 90/60 mmHg, respiratory rate 20 rpm, and oxygen saturation 99% on room air. PE also reveals muffled heart sounds and distended neck veins. He does not respond to aggressive fluid resuscitation. Which of the following is classically true regarding the acute hemodynamic condition affecting this patient? a. The patient should be started immediately on therapeutic heparin b. A finding of an increase in arterial pressure during inspiration > 10 mm Hg would be indicative of this condition c. Chest radiograph is the gold standard in diagnosis d. EKG may show electrical alternans e. Cardiac catheterization would show higher pulmonary capillary wedge pressure than right ventricular pressure during diastole

d. EKG may show electrical alternans - suffering from cardiac tamponade

A 14-year-old patient presents to the ED with an intractable nosebleed. Pinching of the nose has failed to stop the bleed. In the ED a topical vasoconstrictor is tried but also fails to stop the bleeding. What is the appropriate next step? a. Observation b. Anterior nasal packing c. Posterior nasal packing d. Electrocautery of Kiesselbach's plexus e. Electrocautery of Woodruff's plexus

d. Electrocautery of Kiesselbach's plexus

A 57-year-old man presents to general medical clinic with hematuria. He first noticed the color of his urine darkening several weeks ago, and it has recently worsened. He denies any pain. Vital signs are stable. PE is within normal limits. He has no costovertebral angle tenderness. UA is positive for heme and urine culture is negative. Urine cytology reveals malignant cells. Biopsy confirms this is the most common tumor of the genitourinary tract. All of the following are risk factors for this tumor EXCEPT? a. Cyclophosphamide b. Smoking c. Aniline Dyes d. HIV e. Phenacetin

d. HIV - not a risk factor

A 24-year-old male presents with 6 months of auditory hallucinations, paranoid delusions, and gross disorganization. The patient's head CT is shown. Which of the following would favor a good prognosis for this man's condition? a. Insidious onset b. Prior psychiatric history c. Negative symptom predominance d. History of a precipitating event e. Family history of mental illness

d. History of a precipitating event - this indicates a better outcome

A 37-year-old woman complains of rapid-onset mid-epigastric pain with radiation to the back. She states that she has had a few of these episodes in the past, but this is the worst so far. Her pain is somewhat relieved by sitting forward. PE notes epigastric tenderness without guarding or rebound. Also noticed on physical examination is shown. Laboratory studies show elevated amylase and lipase. She denies use of alcohol and tobacco. A RUQ ultrasound was performed and found to be negative and total and direct bilirubin are normal. What is the most likely cause of her current symptoms? a. Alcoholism b. Scorpion sting c. Biliary tract obstruction d. Hypertriglyceridemia e. Hypercalcemia

d. Hypertriglyceridemia - Should be very high on the differential when a patient denies alcohol use, there is no evidence of cholelithiasis, and has xanthomas on physical examination

A 61-year-old female with a 30-pack-year smoking history presents to your office for sinusitis for the 5th time in the last year. Cigarette smoking increases the risk for chronic sinusitis by which of the following mechanisms? a. Immunosuppression b. Cell necrosis in the epithelium lining the sinus c. Type I hypersensitivity d. Impairment of ciliary function e. Thickening of the nasal mucous

d. Impairment of ciliary function

A 20-year-old male presents to his primary care physician for a sports physical. If the physician is worried about hypertrophic cardiomyopathy in this patient, what physical examination finding and hereditary pattern would be expected for this disease process? a. No change in murmur intensity with valsalva; X-linked recessive b. Decreased murmur intensity with valsalva; X-linked recessive c. Decreased murmur intensity with valsalva; autosomal dominant d. Increased murmur intensity with valsalva; autosomal dominant e. Increased murmur intensity with valsalva; X-linked recessive

d. Increased murmur intensity with valsalva; autosomal dominant

A 78-year-old woman presents to the ED complaining of generalized pain and edema in her left leg. She denies chest pain, shortness of breath, and hemoptysis. She takes 2.5 mg warfarin daily for atrial fibrillation and her INR is 2.6. Lower extremity ultrasound reveals the presence of a deep venous thrombosis in the left popliteal vein. Which of the following should be ordered next in the treatment of this patient? a. Increase warfarin dose b. CT angiogram of the chest c. Ventilation/perfusion lung scan d. Inferior vena cava filter e. IV heparin

d. Inferior vena cava filter - already on therapeutic dose of anticoagulant

A 25-year-old female is brought to the ED with an acute asthma exacerbation. The patient takes combination fluticasone proprionate and salmeterol for long-term management of severe asthma. Her symptoms are well-controlled while on medication, but for the past few days she has not taken her medication after she left her inhaler at her tennis club. Her pulse is 100/min and respirations are 26/min. On PE, she is using accessory muscles of respiration. Ronchi and wheezing are present in both lung fields. Oxygen saturation is 85% on room air. The patient is started on 2L/min nasal cannula oxygen. Which of the following is the most appropriate next step in management? a. Systemic corticosteroids b. Oral montelukast c. IV magnesium sulfate d. Inhaled albuterol e. IV theophylline

d. Inhaled albuterol - next appropriate step after supplemental oxygen is inhaled short-acting beta-2 agonist (albuterol, levabuterol,

A 68-year-old male with a history of DM presents to the ED with severe right ear pain. He describes the pain as worse at night. This morning he awoke with discharge on his pillow. On PE, the patient is febrile and looks ill. PE of the ear reveals a finding similar to this shown. What is most appropriate therapy in this patient? a. Ciprofloxacin otic drops b. Vancomycin otic drops c. Oral amoxicillin d. Intravenous ciprofloxacin e. Intravenous vancomycin

d. Intravenous ciprofloxacin

An 86-year-old man with hypertension and hyperlipidemia presents to the emergency department complaining of abdominal pain that has persisted for the past three days. He denies ever having had abdominal surgery. His wife reports that he has not been able to keep any food down and has not had anything to eat or drink in more than 24 hours. An abdominal film is obtained What is the most likely diagnosis? a. Diverticulitis b. Opiate ileus c. Bowel perforation d. Malignancy e. Lyme disease

d. Malignancy - no prior history of surgery

A 24-year-old female is referred to a pulmonologist for worsening symptoms of asthma. Her PMH is otherwise unremarkable except for a worsening of her asthma symptoms during her menses. She describes her period as lasting for several days with severe abdominal cramping that sometimes requires her to stay home from work. If this finding was truly related to her asthma, what would you also expect to find on this patient's physical exam? a. Cafe-au-lait spots b. Skin telangectasias c. Saddle nose deformity d. Nasal polyps e. Increased jugular venous distention

d. Nasal polyps

A 31-year-old man is brought to the ED via ambulance status-post a grand mal seizure, the first of his life. His wife states that earlier in the day he was complaining of a terrible headache, but attributed it to too much time in the sun at the beach. He vomited once prior to the seizure. His PMH is unremarkable. Vitals are BP 160/90 mmHg, temperature 100.9 degrees F, pulse 110/min, respirations 20/min. On exam, he is lethargic and not oriented, has nuchal rigidity and a positive Kernig's sign. Non-contrast brain CT appears as seen. What is an important component of treatment for this patient? a. Observation b. Lowering the head of the bed c. Permissive hypertension d. Nimodipine e. Nitroprusside

d. Nimodipine - Vasospasm, secondary to the inflammatory response, is an important complication of ruptured aneurysms

A 9-year-old girl is brought to the pediatrician by her mother who is concerned about a new rash (shown). Her daughter is otherwise feeling well. Which of the following is to blame for this patient's condition? a. Human herpes virus 6 b. Human herpes virus 8 c. Paramyxovirus d. Poxvirus e. Parvovirus

d. Poxvirus

47-year-old gentleman presents to the emergency department with a 3-4 day history of cramping abdominal pain, nausea, and vomiting. PE reveals a normal cardiovascular assessment, but his abdomen is found to be distended, tender to palpation, and with high-pitched bowel sounds. An abdominal radiograph is shown. Upon further questioning, what is most likely to be found in this patient's past history? a. Psychological derangements b. Gastroesophageal reflux disease c. Irritable bowel syndrome d. Previous abdominal surgeries e. Intermittent appendicitis

d. Previous abdominal surgeries - adhesions are most common cause

A 5-month-old male infant presents to your office in January. His mother complains that his cheeks are "red and scaly." She's concerned that they are uncomfortable, since she's noticed him scratching them. The rash has worsened since winter started. Otherwise, he has no medical history. Family history is relevant for asthma and some seasonal allergies. On exam, his cheeks bilaterally are erythematous with erosion, scaling and excoriated papules and plaques. Small, similar appearing patches are also present on his chest. What is the most likely outcome for this condition? a. Spontaneous resolution in 1 month b. Resolution with steroids in 1 month c. Resolution with antibiotics in 1 month d. Recurrence, regardless of treatment e. Recurrence, lessened by antibiotics

d. Recurrence, regardless of treatment

A 55-year-old female was admitted to the hospital after a motor vehicle accident. A family member mentioned to the nursing staff that the patient had been chronically taking a medication for her anxiety. If by the third day of her hospitalization, the patient has not been restarted on her home medication regimen, what clinical manifestations could be experienced by her? a. Excessive somnolence, low heart rate, and low blood pressure b. Fever, tachycardia, HTN, tremor, elevated CPK, "lead pipe" rigidity c. Dry mouth, blurred vision, urinary retention. d. Restlessness, confusion, tremulousness, psychosis, and generalized seizure e. Darting and writhing of face, tongue, or head

d. Restlessness, confusion, tremulousness, psychosis, and generalized seizure - all symptoms consistent with withdrawal from benzodiazepines

A 52-year-old G3P3 woman with a BMI of 24 presents to her physician complaining of "wetting" herself. She reports a light leak of urine whenever she coughs or laughs. She has had this problem now for several years. Upon the recommendation of a friend, she has been trying to strengthen her pelvic floor muscles by performing Kegel exercises. She has been performing these regularly for the last 2 weeks, but has had no improvement in her symptoms. She has no other complaints. Which is the appropriate recommendation? a. She will need a urethropexy b. She should lose weight c. She should have a trial of antibiotics d. She should continue Kegel exercises for a total of 3 months e. There is no efficacious way to treat these symptoms

d. She should continue Kegel exercises for a total of 3 months

62-year-old white male presents to his PCP with concern for weight gain, abdominal distension, and breast enlargement. Physical exam reveals an overweight male with bilateral gynecomastia and a distended abdomen with evidence of shifting dullness. You also note several skin lesions. Which aspect of the patient's history would reveal the most-likely underlying cause of the patient's chief complaints? a. Surgical history revealing a blood transfusion in 1984 b. Travel history revealing time spent 20 years ago in South America c. Review of systems revealing constant headaches and nausea that are worse upon waking d. Social history revealing alcohol abuse e. Medical history revealing arthritis self-treated with large quantities of acetaminophen

d. Social history revealing alcohol abuse - this is classic for cirrhosis

A 60-year-old woman presents with diffuse colicky abdominal pain, nausea and vomiting. The patient's past medical history is significant for hypertension and perforated appendicitis at age 25 that was managed surgically. Her physical exam reveals hyperactive bowel sounds, diffuse abdominal tenderness and distention. An abdominal x-ray is shown below. Which of the following additional features would result in the patient being managed surgically? a. The patient has not passed gas b. The patient has a reducible ventral hernia c. The patient is volume depleted d. The patient has rebound tenderness e. The patient is a diabetic

d. The patient has rebound tenderness - sign of peritonitis and absolute indication for surgery

A 6-year-old boy presents to general pediatrics clinic with a rash. He is in an outdoor club and was camping with his father several days ago. He has no other complaints and no past medical history. He does not take any medications. His vital signs are stable. His PE is significant for the following rash as shown localized to his right forearm. The pathophysiology of this condition is described as a: a. Type I hypersensitivity reaction b. Type II hypersensitivity reaction c. Type III hypersensitivity reaction d. Type IV hypersensitivity reaction e. This is not a hypersensitivity reaction

d. Type IV hypersensitivity reaction

A 29-year-old female presents to general medical clinic with dysphagia. Her symptoms began several months ago. She has trouble swallowing solids and liquids though liquids seem to make her choke and sputter the most; therefore, she has been unable to eat and has thus experienced significant weight loss. She has a 20-pack-year smoking history. She denies any recent travel. VSS. PE is within normal limits. A barium esophagram shows the following . Subsequent esophageal manometry reveals elevated resting lower esophageal sphincter pressure, incomplete lower esophageal sphincter relaxation after swallowing, and almost total absence of peristalsis in the esophageal body. What is the next best step in management? a. Begin a calcium channel blocker b. Begin botulinum toxin injections c. Endoscopic balloon dilation of the lower esophageal sphincter d. Upper endoscopy e. Myotomy with fundoplication

d. Upper endoscopy - must rule out malignancy

A 31-year-old G3P2 presents for a first-trimester prenatal visit. You perform the standard first-visit tests including blood type and screen, Rh type, CBC, immunization status, urinalysis for protein and glucose, and Pap smear. Which of the following first-visit tests will you also administer in the third trimester? a. Blood type and screen b. CBC c. Immunization status d. Urinalysis e. Pap smear

d. Urinalysis - all of the other choices are during the first prenatal visit

A 75-year-old alcoholic man with a history of multiple sclerosis presents to the ED with self-described fevers and chills over the past 24 hours. He is concerned because he has no recollection of how he got to the park bench upon which he awoke yesterday morning. A chest x-ray is obtained. In patients with this illness, which of the following is LEAST likely implicated in its development? a. Neurologic dysphagia b. Alcohol-induced altered consciousness c. Mechanical obstruction of the glottis d. Viral infection e. Multiple sclerosis

d. Viral infection - this is aspiration pneumonia secondary to impaired consciousness due to his excessive alcohol use

A 36-year-old male who recently emigrated from Southeast Asia was treated several weeks ago at a local hospital for pulmonary tuberculosis. He was started on therapy including rifampin, isoniazid, pyrazinamide, and ethambutol. He now reports numbness in all of his extremities which began one week ago and has been progressing. On PE he has an uncoordinated gait and paresthesias on his hands and feet. What is the most likely cause of his symptoms? a. Vitamin A deficiency b. Vitamin B1 deficiency c. Vitamin B3 deficiency d. Vitamin B6 deficiency e. Vitamin B12 deficiency

d. Vitamin B6 deficiency

A 1-day-old male infant born by normal spontaneous vaginal delivery was found to have a scrotal mass on physical examination. The mass transilluminates and confirms the attending physician's suspicion of hydrocele. How long should the physician recommend that the parents wait before the hydrocele is reassessed? a. 1 day b. 1 week c. 1 month d. 6 months e. 1 year

e. 1 year

A 61-year-old male visits a medical student clinic booth during a health and wellness fair at his workplace. A medical student listens to his heart and lungs and then immediately asks the man if he's ever been diagnosed with HTN. Which of the following concerning findings might have led her to suspect this man's hypertension? a. A crescendo-decrescendo murmur radiating to the carotids b. A holosystolic murmur radiating to the axilla c. A rough, scratching sound d. A dull, low-pitched sound early in diastole best heard with the bell e. An extra heart sound heard immediately before S1

e. An extra heart sound heard immediately before S1 - long-standing HTN causes left ventricular hypertrophy and may result in a stiffening of the left ventricle and production of an S4 heart sound, an extra sound heard immediately before S1

A 30-year-old female is brought to the ED by her husband after refusing to sleep for a week and spending the couple's entire savings account on a collection of fur coats. She refuses a PE and insists that she wear a black sequin dress rather than a hospital gown. For several hours, the patient is overcome by fits of laughter and refuses to answer questions. She then insists that she be allowed to leave the hospital immediately. Urine toxicology is negative. Which class of drugs should be administered? a. Selective serotonin reuptake inhibitor (SSRI) b. Serotonin-norepinephrine reuptake inhibitor (SNRI) c. Anticonvulsant d. Mood stabilizer e. Antipsychotic

e. Antipsychotic - patient is agitated

A 71-year-old man presents to a dermatologist concerned that he has "skin cancer." He was watching a television report about skin cancer awareness and is now concerned that a "mole" he has on his chest is cancer. On dermatologic exam, the lesion is similar to that in the figure. What is the most appropriate next step in the management of this patient's "mole"? a. Incisional biopsy b. Excisional biopsy c. Cryoablation d. Mohs microsurgery e. No therapy

e. No therapy - no thereapy at this time - this is a benign lesion

A 24-year-old G1P1001 female presents with complaints of facial acne and lack of menses for the past 7 months. She does not currently desire to become pregnant. Her VS are as follows: T 37.3 C, BP 135/76, HR 74, RR 14, O2 Sat 99%, BMI 34.5. PE is significant for comedonal acne across her forehead and chin as well as the presence of terminal hairs on her upper lip, chin, and chest between her breasts. TSH and prolactin levels are normal. Serum testosterone level is elevated and fasting blood glucose level is 135 mg/dL. Which of the following pharmacologic treatment regimens would be the most appropriate therapy for this patient? a. Leuprolide b. Combined oral contraceptives c. Combined oral contraceptives and clomiphene citrate d. Finasteride e. Combined oral contraceptives and metformin

e. Combined oral contraceptives and metformin - this is PCOS

A 27-year-old female presents to your office for a check up. The patient is currently on paroxetine, though denies depressive symptoms for the last 6 months. She has had 2 episodes of major depression. How do you proceed with this patient? a. Discontinue paroxetine today b. Continue for another 6 months, and if symptom free, discontinue paroxetine c. Continue for another 12 months, and if symptom free, discontinue paroxetine d. Continue for another 3 years, and if symptom free, discontinue paroxetine e. Continue paroxetine indefinitely

e. Continue paroxetine indefinitely - history of 2 episodes

A 26-year-old first-year medical student with a medical history significant for GERD comes to your office because of frequent episodes of palpitations which are sudden in onset and accompanied by sweating and a sense that she is going to "pass out." The episodes typically last no more than 10 minutes, and although the patient feels as if she may syncopize, she never has. The episodes first appeared when she started medical school and have increased in frequency to the point where she is afraid to attend lectures out of fear of having an "attack." What is frequently associated with this patient's condition? a. Irritable bowel syndrome b. Fibromyalgia c. Somatization disorder b. Conversion disorder e. Depression

e. Depression

A 45-year-old female undergoes a transphenoidal approach for a pituitary prolactinoma. Surgery proceeded without complications and the entire mass was removed. The patient's urine output is 4 L on post-operative day 1, and labs are significant for serum Na of 145 mEq/L (normal: 135-145). Urine osmolality is 185 mOsm/kg, and urine specific gravity is 1.004 (normal: 1.012 to 1.030). Which of the following choices is the next best step? a. Water restriction b. Loop diuretic c. CT scan of the brain d. 0.45% NaCl administered intravenously e. Desmopressin

e. Desmopressin - this is central DI from surgery

A 61-year-old female presents with several episodes of feeling "dizzy." She reports that these episodes were associated with a "ringing" in her right ear as well as sense of hearing loss. She also describes a sensation of increased "fullness" on the affected side. Past medical history is significant for osteoporosis for which she takes ibandronate and hyperlipidemia for which she takes atorvastatin. Which of the following is the first-line treatment for this patient? a. Acetazolaimde b. Meclazine c. Intraaural gentamicin injections d. Endolymphatic sac drainage e. Dietary changes

e. Dietary changes

A prison guard working at a state penitentiary has a new 24-year-old male inmate come to the cell block that he patrols. This new inmate has a diagnosis of antisocial personality disorder. What types of behavior (past or present) can the prison guard expect from this inmate? a. Pattern of unstable relationships, poorly formed identity, impulsive, reckless, self-mutilating, and feelings of emptiness b. Sense of superiority, need for admiration, and lack of empathy c. Attention-seeking, excessive emotions, dramatic, flamboyant, sexually inappropriate, and provocative d. Social withdrawal, reclusive, no desire for close relationships, and prefers to be alone. e. Direct violation of the rights of others, aggressive, impulsive, and superficial charm that allows for manipulation of others

e. Direct violation of the rights of others, aggressive, impulsive, and superficial charm that allows for manipulation of others

A 28-year-old female with no PMH presents to your clinic with complaints of several days pale green frothy vaginal discharge. A sample of the discharge reveals pear-shaped motile cells. You diagnose her with Trichomonas vaginalis and prescribe her metronidazole. She states that she has had a single sexual partner and requests treatment for him as well. Which of the following is NOT a pathogen that requires treatment for the partner as well? a. Pediculosis pubis b. Treponema pallidum c. Neisseria gonorrhoeae d. Chlamydia trachomitis e. Gardnerella vaginalis

e. Gardnerella vaginalis - not a sexually transmitted disease

A 62-year-old woman presents to the urgent care clinic complaining of vision changes, headaches, and leg pain while walking that is relieved by rest, which has been progressing over many years. She cannot remember the last time she visited the doctor and she takes no medications. On PE, she is found to have a BP of 175/95. Her basic metabolic panel is as follows: Na 132 K 3.8 Cl 102 HCO3 23 BUN 70 Cr 4.2 Glu 360 The patient is diagnosed with end-stage renal failure and started on dialysis. Screening for which of the following would have been most appropriate to detect this patient's progressing renal failure? a. Hematuria b. Leukocyturia c. Urine nitrites d. Leukocyte esterase e. Microalbuminuria

e. Microalbuminuria

A 57-year-old with a 30-year-history of type I diabetes presents to general medical clinic with a lesion on his foot. Although he was advised to wear orthotics by his podiatrist, he decided to keep wearing his dress shoes and reports that he observed this lesion when his sock was stained with blood yesterday morning. His vital signs are temp of 37 degrees Celsius, heart rate 75/minute, BP 145/90 mmHg, respiratory rate 12/minute, and oxygen saturation 99% on room air. PE reveals a diminished response to the monofilament test. What is the most significant risk factor for development of this condition? a. Peripheral vascular disease b. Female sex c. Duration of diabetes mellitus d. Smoking e. Neuropathy

e. Neuropathy

A 30-year-old otherwise healthy woman presents clinic with a rash. She has no history of atopic dermatitis, other forms of eczema, or asthma. She has no allergies and denies any recent medications. Her vital signs are within normal limits. On PE, there is an erythematous rash on her ears bilaterally. There are no other appreciable skin lesions. Which of the following is the most likely causative agent of this patient's pathology? a. Platinum (Type IV hypersensitivity reaction) b. Gold (Type III hypersensitivity reaction) c. Latex (Type IV hypersensitivity reaction) d. Cosmetics (Type III hypersensitivity reaction) e. Nickel (Type IV hypersensitivity reaction)

e. Nickel (Type IV hypersensitivity reaction)

A 20-year-old college student presents to her PCP for an annual check-up. She is otherwise healthy, but when asked about how classes are going, she expresses serious concern about her upcoming oral presentation for a public-speaking course. She says that she has never been a good speaker in front of other people, and even refrains from asking questions in class because of her nerves. What is the most appropriate treatment to alleviate these symptoms for her upcoming final? a. Cognitive behavioral therapy b. Relaxation exercises c. Fluoxetine d. Amitryptyline e. Propanolol

e. Propanolol

A 14-year-old boy is accompanied by his mother to general pediatrics clinic for a routine visit. The boy has no complaints. He is doing well in school and has no medical illnesses. However, his mother reports that she is concerned about the growth in his chest. She states that over the past several months he has been developing what seem to be breasts. VSS. PE reveals left chest slightly larger than the right breast. Both are soft to palpation and nontender. What is the next step in management? a. Order serum testosterone b. Order serum estrogens c. Order a mammogram of both breasts d. Biopsy the left, larger breast e. Reassurance

e. Reassurance - gynecomastia is seen in two-thirds of adolescent boys during mid to late puberty

A 68-year-old male presents to his PCP for a routine health maintenance examination. Past medical history is unremarkable and the patient does not take any medications. Physical exam demonstrates a crescendo-decrescendo systolic ejection murmur at the right second intercostal space that radiates to the carotids. The patient does not complain of chest pain or difficulty breathing. EKG is suggestive of LV hypertrophy. The patient is followed with serial echocardiography. Three years later, the patient comes back to the physician's office complaining of chest pain with exertion. What is the most appropriate management of the patient at this time? a. Continue to follow him with serial echocardiography b. Follow him closer with shorter lengths of time between appointments c. Have him follow up on an as-needed basis d. Prescribe nitroglycerin tablets for his exertional angina e. Refer him to a cardiac surgeon

e. Refer him to a cardiac surgeon - when AS becomes symptomatic the aortic valve must be replaced

A 22-year-old male is brought to a psychiatrist by his mother, who claims that he has refused to bathe for two weeks and has not shaven for several months. The patient states that he refrains from grooming because he is sure such practices will help him win the lottery. This behavior is most typical of which of the following personality disorders? a. Obsessive-compulsive b. Schizoid c. Avoidant d. Dependent e. Schizotypal

e. Schizotypal - eccentric in appearance, have odd beliefs, and have trouble developing interpersonal relationships

A 29-year-old computer programmer presents to the general medicine clinic complaining of dizziness, fatigue, and shortness of breath. One month ago, the patient went on a camping trip in northern Connecticut. On assessment, vital signs are temperature 36.9°C, blood pressure 100/65, heart rate 41, and respiratory rate 16 with an oxygen saturation of 99%. A print of the EKG strip is shown below. What is the most likely diagnosis? a. Wolff-Parkinson-White syndrome b. Second degree Mobitz type I AV block c. Second degree Mobitz type II AV block d. First degree AV block e. Third degree (complete) AV block

e. Third degree (complete) AV block - atria and ventricles beat independently of each other because of disruption in the conduction pathway in the heart

A 70-year-old male presents to his PCP with complaints of urinary frequency, urgency and nocturia. Physical exam shows an enlarged, rubbery, non-tender and firm prostate without discrete nodules or masses. Which of the following treatments for this condition is matched with its most common side effect? a. Prazosin - Hypertension b. Finasteride - Hair loss c. Finasteride - Increased size of prostrate d. Transurethral electrovaporization (TUEVP) - Cancer e. Transurethral resection of the prostate (TURP) procedure - Retrograde ejaculation

e. Transurethral resection of the prostate (TURP) procedure - Retrograde ejaculation

A 32-year-old patient who is approximately 9 weeks gestation presents to your clinic complaining of a rash on her thigh that appeared yesterday. She has no other symptoms, and PE does not reveal any abnormalities other than the rash. She recently returned from camping in Wisconsin 5 days earlier. Her history is significant only for Lyme disease 2 years earlier, and an episode of streptococcal pharyngitis that was treated with penicillin, to which she had an anaphylactic response. What is the best next course of action? a. Assure her that the rash will resolve on its own b. Send IgG levels for Borrelia burgdorferi c. Treat with 10 days of amoxicillin d. Treat with 10 days of doxycycline e. Treat with 10 days of erythromycin

e. Treat with 10 days of erythromycin - patient is allergic to penicillin

A 56-year-old woman presents to her primary care physician complaining of heartburn, belching, and epigastric pain that is aggravated by coffee and fatty foods. She states that she has recently been having difficulty swallowing in addition to her usual symptoms. What is the most appropriate next step in management of this patient? a. Trial of a proton pump inhibitor b. Nissen fundoplication c. Lifestyle changes - don't lie down after eating; avoid spicy foods; eat small servings d. Trial of an H2 receptor antagonist e. Upper endoscopy

e. Upper endoscopy - indicated with her symptoms


Related study sets

Daily Activity versus Planned Exercise

View Set

Microeconomics Chapter 1-3 Quizzes (add 4 after sunday)

View Set

Changes in Period and Phase Shift of Sine and Cosine Functions

View Set

I/O Psych fill in the blanks, I/O Psych Final 3/3, I/O Psych Final 1/3, Industrial Organizational Psychology Module 5, I/O Chapter 15, I/O Chapter 14, I/O Chapter 13, I/O Chapter 12, I/O Chapter 11, I/O Chapter 10, I/O Chapter 9, I/O Chapter 8, I/O C...

View Set

Vocabulary Workshop Level H Unit 1

View Set

Chapter 2 - Securities, Markets, and Transactions

View Set